is my first job supposed to be crappy?

A reader writes:

I have a question about job searching as a new grad. I’m currently in the interview process for a job I’m really not so sure about. I haven’t received an offer or anything, but I wanted to ask a question about the standards I should have for my first job out of college just in case.

I’ve researched the company, and based on salaries people have disclosed publicly, the position I’m in the running for pays much less than market value. I’m also pretty sure it’s a fair amount less than I’m worth, since my technical field of study provided me with a lot of hard skills. Most importantly, I’m concerned the pay might not be enough for me to live on. The public reviews also make me worried about the office culture.

Should this company offer the job to me, my parents say I should take it. They argue that as a young single person with minimal debt (I only have student loans), I would be able to make a low salary work. I’ve been out of the workforce since graduation recovering from a mental health issue (all better now!), so they say it would be good just to get something on my resume on this point, even if I have to sacrifice a comfortable living situation for it. Finally, the job is in the location I’m looking to move to, so they say it would be worth taking it to get established in the area.

From my perspective, this looks like a bad job. The pay isn’t fair, people seem overworked, and it sounds like there’s a lot of petty office politics at play. I’m actually sort of worried that a bad job paired with bad living conditions would exacerbate my currently stable and well-managed mental health issues. But as someone just getting into the working world, I’m not sure what I should really expect from my first job. Is it worth it to just take the first crappy job I’m offered and try to jump ship as soon as possible? Is my first job just supposed to be crappy, or is this how people get taken advantage of?

It’s true that most first jobs aren’t super awesome. They’re often low paid (often less than you think you’re worth) and involve not-terribly exciting work, and you usually don’t have the standing to be very picky yet.

But there are some things that go beyond the typical dues-paying of a first job. You shouldn’t expect to have to put up with pay that’s wildly of out sync with market rates, labor law violations, or an abusive environment, no matter how inexperienced you are.

The problem is that I can’t tell whether any of those things are in play with this job.

Let’s take salary first. When you evaluate salary, you need to look at the market rate for this particular type of job at this particular level (so entry-level, not what they’d pay someone in a similar role but with more experience) in this particular geographic area and often in this particular field. You can’t base it on things like “my technical field of study provided me with a lot of hard skills.” Market rate is what counts. So, if you haven’t already, the first thing to do is to get a really solid understanding of the market rate, taking all those factors into account.

When you do that, you might find out that the amount you can expect to earn right now doesn’t line up with the amount you think you’ll need to live on. (Hopefully not, but it could happen.) If that’s the case, you have a few options: You can see if you’re qualified for different types of jobs that pay more — possibly in other fields, in which case you’d have to decide if you’re willing to make that trade-off … or you can see if you can cut expenses, like by living at home, getting roommates, etc. … or you can decide you’re willing to work a part-time job in addition to the main job (I think that’s tough to do, but lots of people do it).

Now, office culture. People being overworked isn’t terribly unusual, but whether it should be a deal-breaker depends on what that looks like in practice. “We never have time to get everything done” isn’t outrageous in a lot of fields. “We’re expected to work 70-hour weeks” generally is (with some exceptions where you know up-front that insane work hours are part of the deal, like big law). So details matter here. The same is true of “it sounds like there’s a lot of petty office politics at play” — you’ll find office politics everywhere you go, so that on its own isn’t prohibitive, but the details will matter. (And complicating this is that as someone new to the work world, you don’t yet have a finely tuned barometer for what’s a big deal and what isn’t. Read this.)

The really important thing to factor in here is what your other options are. If you’re easily getting interviews and are confident that you can get other offers, it makes sense to walk away from a job that you have serious doubts about. But if you’ve been searching for a while and this is the only offer that’s materialized, that might be the market telling you that you’ll need to take it in order to build up a work history that eventually will give you more options. Sometimes you have to go with something not ideal in order to get the experience that will help you get a better job later.

Your parents aren’t off-base* when they say that since you haven’t worked since graduation, it would be good to get something on your resume now. That’s a real concern, because the longer you go without a job after graduation, the harder it will get to get a good one. Again, that brings us back to the question of what type of options you have. If this is the only one, I’d lean toward agreeing with them, unless you think the job will cause you real harm.

* possibly the first time I have written those words here

Read an update to this letter here.

{ 211 comments… read them below }

  1. vic*

    You can do what I did: low-paying nonprofit job where the boss has basically checked out and lets you handle things, which gives you a ton of responsibility and a lot to put on your resume.

    1. AMG*

      Yes. I had an awesome low-paying job in college that I took for the experience and then waited tables to supplement my income. It put me waaay ahead of my peers and I was a shoe-in for consulting at the top 5 firm of my choice.

      But don’t think it’s absolutely going to suck. I interviewed for my current position in the ugliest, most depressing building you could think of. Horrible vibe. It has been my most challenging, high-paying, rewarding job ever and I love it. You can’t necessarily tell from the outside what it will be like.

    2. esra*

      Hands up for low-paying nonprofit jobs with disinterested bosses. That’s what gave me my start as well. A bit scary, but if you dive into the challenge, it can really pay off.

        1. Victoria Nonprofit (USA)*

          Me three — although it was less that my boss was disinterested, and more that there were just four of us to do everything, so even little no-experience me got to/had to do lots of interesting and hard stuff!

        2. Princess Consuela Banana Hammock*

          Same! Although having checked-out bosses was not awesome for gaining a sense of what are reasonable professional norms, it gave me a LOT of room to grow and learn and be a little audacious (I definitely got away with things which, in retrospect, make me cringe a little).

          But also note, OP, that nonprofit salaries are almost always below market. So when you’re evaluating affordability, you have to measure against other comparable nonprofits in your region/area.

      1. Overeducated*

        I wanted to do that after grad school, but unfortunately the pay was too low and hours too intense to make it work by the time I had a family :( this would not have been with a disinterested boss , but lots of responsibility!

      2. CodingGrrl*

        Yes, same here! My first job was a contract data entry job working out of the company owners’ apartment (they were a very smart, very eccentric married couple). I was paid a very low hourly wage and often didn’t have a lot of work that I could bill and thus get paid for. Looking back, it was a pretty shady setup. However they taught me programming for some work tasks, and we also got free lunch every day. I ended up staying 2 years, at the end of which I landed my current awesome web developer job with full benefits and good pay. I look back on that time as almost the equivalent of a degree in computer science. It definitely skyrocketed my marketability!

      3. Stephanie*

        Yeah, that was my last job before I came back to grad school. My boss was basically collecting a check until retirement, so I was able to accomplish a decent amount while I was there. It sucked during it, I’ll be honest, but the job does look good on my resume

        1. esra (also a Canadian)*

          Exactly. It can be a total grind, during. And working for a checked out boss is crummy, but I got projects most junior designers wouldn’t be allowed in the same room with. I sucked it up for a couple years, then moved onto better with a few great pieces in my portfolio.

      4. Bluebell*

        Same here. And after boss left I was named acting, but of course without a raise. I never made that mistake again.

      5. Kj*

        Yep. I was thrown into the deep end, my team lead didn’t lead and my big boss was too busy. I got handed a list of clients and was told to figure it out. Three years later, I’m well respected and can name my price in some areas.

    3. Emmie*

      I started at a small business. Less rigid job descriptions, frequent ability to take things on, and lots of interaction with the owner. Benefits were horrible and stability wasn’t there. But, the experience paid dividends in my ability to get later jobs.

      1. Ama*

        Me too. I was (as I don’t think either I or my bosses realized at the time) illegally classified as an independent contractor and had no benefits at all, but I ran the entire office for a family therapist, learned a lot of handy skills about dealing with insurance companies, setting office policy (and, although I didn’t realize it at the time, setting boundaries with my employers when the found the line between “office manager” and “personal assistant” a little fuzzy). But that job helped me get a better-paying student job when I went back to grad school (the woman I first interviewed with looked at my resume, said “oh you’re way too qualified for this job, but this other department needs a part-time secretary, let me send them this”, that grad school job made it easier to get into full-time academic administration, and so on.

        I’ll admit though that I only survived that job because I was living at home, had access to affordable health insurance through other means, and knew I had a firm end date because I was going to grad school.

    4. Koko*

      I had one of those. And when I was hiring my replacement, I actually said more or less as much in the job ad. Something like, “This is a great opportunity for someone new to the field who is ready to lead a growing program and make it your own.” Basically a more diplomatic way of saying, “We can’t afford someone with experience, but you’re going to have the kind of total autonomy that you usually don’t get until you have experience.”

    5. TCO*

      I had similar experiences at my first nonprofit jobs, except that I mostly had really good bosses who weren’t checked out! There was just so much work to be done and they couldn’t afford a lot of middle management (or even seasoned individual contributors) so the people at the bottom had lots of room to take on new tasks and grow. In my late 20s, it was actually a shock to end up in a top-heavy, well-funded workplace who is paying me way more to have way less responsibility. Even after a few years here I don’t have the level of responsibility and autonomy I had earlier in my career (and I really miss that, but the pay here is nice).

    6. Rat Racer*

      I started at a huge white collar criminal defense firm in DC as a paralegal, a job I accepted from a pay phone in Italy while backpacking with my sister. It was probably not a terrible job, but I was terrible at it and miserable. Yet, I would totally do it again for the following reasons:
      1. It paid the bills and put something on my resume while I hunted for something more compelling
      2. It taught me a LOT about professional norms (which I learned by breaking them and getting my hand slapped)
      3. It taught me the importance of demonstrating competence at administrative tasks. Arrogant little twerp that I was coming out of undergrad, I couldn’t believe that I was deploying my fancy pants ivy league degree toward xeroxing and bates stamping, and had the attention of a gnat when it came to detail. I learned an important lesson: if you can’t demonstrate competence at the tasks you’re given – no matter how simple – no one will trust to you take on anything bigger. Oh yeah, and they will probably fire you.

      1. sstabeler*

        true- while I generally disagree with the idea that entry-level jobs should not pay enough to live on ( that’s a separate rant) it’s true you don’t have much choice as the entry-level employee.

  2. SouthernLadybug*

    First a “no this is certainly NOT legal” conversation yesterday and now we agree with parents! It’s the coming of the apocalypse :)

    Seriously though, I am with Alison on this answer. It’s hard to say without more information, which would be identifying. For example, in my social service field, jobs with a state agency usually are underpaid, people are overworked and sometimes (depending on department) the culture isn’t what it could be. But, with a few exceptions, an entry level job there is what provides the experience for better paying ones available in the area. So if you were a recent grad from the program I sometimes teach at asking for help, in most cases I would say take the job for a year or two if it’s your only option.

    I also know of another org that the same descriptors could be used for….and I’d tell that same graduate to avoid it if possible.

  3. Leatherwings*

    I feel like nearly everyone I know took a crappy first or second job to get ahead. I certainly did. I think the most important thing is that the not-so-amazing job at least gives you the opportunity to grow and develop skills that will be helpful when applying for a less crappy job a year or two later.

    1. EddieSherbert*

      That was my experience as well – though, even for ToxicJob, there were parts of it I enjoyed.

      And honestly, I started at a company in a job I wasn’t excited about… but within a relatively short time the company moved me into a position that was perfect! It still ended up being a bad fit for the company culture, but I think my career would have gone very differently (in a bad way) if I hadn’t taken a chance on that job.

    2. Jesmlet*

      And having a crappy first job makes your (hopefully) better second job seem much better. There are a lot of important life skills you can learn working in a less than perfect environment. If you can financially make it work, I would take the job. The longer you wait after graduating, the worse this’ll look on your resume.

      1. Leatherwings*

        Absolutely true. Years later I am still forever grateful for nights and weekends after I didn’t have any for awhile.

      2. Alton*

        I agree with the caveat that you have to be careful sometimes not to let a bad situation get to you or poison your perspective in ways you may not be aware of. I didn’t realize how much a bad job that lasted one month had screwed with my sense of perspective until I spent several months at my new job convinced that I’d be fired over the smallest mistake. Learning to deal with conflict and less than ideal circumstances is good, but you have to actively develop those skills.

        1. Julia*

          This is my worry for the OP as well. Crappy Job to start out with, sure. Toxic Job at Dysfunctional Inc. might leave her with weird ideas about work that may carry over to the next job.

    3. sunny-dee*

      And not just developing skills — it’ll help her develop a network of people who themselves will likely be moving around and so can open doors to future jobs. (LinkedIn is your friend here.)

      If there were a different reason for the gap between graduation and work — like grad school, mission or volunteer work, traveling Europe — that may be less important simply because those other things themselves help with meeting people and making connections, so you can use your network to help find a first job. But with it being a medical reason, more than likely the OP doesn’t have a professional network started yet, so it is really important to kick that off.

    4. Sunflower*

      True. My first job paid super crappy and I hated our company president (so misogynistic). But I generally liked the people I worked with and there’s no way I would have gotten my (pretty great!) job now if I hadn’t worked there.

      You can always take the job and start applying. I started applying right after I took the job and it still took me 2.5 years to get my current one.

    5. Anansi*

      Definitely. My first job was an absolute disaster – terrible pay, dysfunctional office, horrible boss, the works. But looking back on it it definitely had some positives. Surviving on such a small salary taught me how to budget and be organized and frugal with my money (plus it made my next job, which was still slightly below market value, seem like a windfall). Working with a bunch of people who were terrible to each other gave me a crash course in handling difficult colleagues and workplace politics. And having a boss who didn’t even come to work most days meant that I basically ran an entire department at the age of 22, and the skills I learned from that sure came in handy when I was job searching.

  4. Murphy*

    Your first job may not be awesome or ideal, but it shouldn’t be outright crappy…problem is that you don’t necessarily know whether it falls into the outright crappy category until you’re there. This is a tough one. I understand the need to take what you can get (within reason) but I also understand not wanting to accept a job you already have serious reservations about.

    1. BRR*

      I so agree with all of this. It’s not that first jobs are inherently crappy, I think that they have garnered that reputation because there’s a higher chance the work will be mundane (and almost all jobs have mundane parts).

      1. Jessesgirl72*

        And the people taking them don’t have the experience yet to know their expectations are unrealistic.

        1. Koko*

          Right – first jobs aren’t typically crappy, but crappy jobs are typically first jobs because people with experience don’t go near them.

      2. Mike C.*

        Let’s not forget the sorts of businesses that tend to hire younger/newer workers because they aren’t familiar with labor laws.

          1. Anxa*

            I’ve seen (and kind of myself fell into) the trap where you work or volunteer part-time for years on end, and then employers (sometimes those very same employers) won’t offer a the holy grail full-time job with benefits because you haven’t paid your dues by working full-time for pay. Of course there are some very good reasons for this and there’s the classic experience catch-22, but I’ve definitely seen it become exploitative.

        1. BRR*

          I see now my comment is not really clear at all. There are unfortunately many employers that prey on their employees. I was thinking of people who say all first jobs are crappy and might shut down people who disagree. First jobs are automatically crappy, they just have a higher rate of not fulfilling everything somebody wants in a job.

  5. Brigitha*

    You’ve probably already thought about this, but I want to offer something that hasn’t been mentioned yet. You mentioned your concern for your mental health. You’re contemplating moving to a new area, presumably away from your current support network, and possibly into a stressful work environment. If you decide to do this, make it a priority to re-establish a support network in your new area. Your first job will probably come with a difficult adjustment period no matter how well-run or dysfunctional the workplace is. If you have a good support system in place, you have a better chance to power through a crappy first job for a year or two until you can move to something better. Good luck.

    1. designbot*

      yeah I think that one of the things that could potentially give in her list of ideals (salary, job type, environment, etc.) is location. It may be worthwhile to take something local to where you are now to get your feet wet and make the move once you have a better resume to work with.

    2. Isben Takes Tea*

      Yes, this! A new job will always come with stress, periods of self-doubt, and other mental health challenges, even for those who haven’t dealt with prior issues. Make sure you have a self-care plan and walk yourself through situations, like, If I’m feeling lonely I will do X/call Y; if I’m self-doubting I’ll do A/call B.

      Also, if you decide to take the job, try to figure out ahead of time what your exit strategy is/your limits are. Is there a line anywhere that you don’t want to cross? (working X hours overtime, being called at home, boss making you cry, etc?) You can even sketch out your plan for pursuing the job after this one, just so you feel prepared.

      Good luck!

    3. GigglyPuff*

      That’s what I was coming to point out. It’s really easy to relapse (on any on-going issue) when you move to a new area, and it can be even harder if your job is a stress factor. So definitely take the time to find people and activities outside work right away. It can be hard, took me a while to even realize I needed to do it, and it’s taking me even longer to actually make myself do it, but I definitely think it’s necessary.

    4. Carpe Librarium*

      Yes! Also, you mentioned your mental health is in a good place now (yay!), but if you haven’t already, speak to your therapist/counsellor about keeping lines of communication open. If you don’t presently have an ongoing plan set up, maybe arrange a semi-annual check up, and if you decide to change location, request a referral or two for a local provider.
      If you find yourself struggling with any aspect of the move/job you will already have a key support ready.

  6. Dankar*

    Thank you, thank you, thank you for posting this today! I just got an offer yesterday and, while I actually think I’ll like the office and the work, the salary isn’t exactly what I wanted it to be. Even though this isn’t my question, your answer has provided some much needed perspective.

    1. Archie Goodwin*

      For what it’s worth, as someone who’s been in the working world for a decade now…salary isn’t the first thing I look at. As long as it’s an amount I can live on (of course), I’m fine with that…I don’t mind taking a lower amount if I like the work and I like the office. Personal choice, I know, but still.

      1. Jessesgirl72*

        As Alison pointed out, though, even the “can I live on it?” question changes with experience.

        I lived with my parents for my first job. My husband put living without roommates as a priority, and really regrets that decision. He racked up quite a bit of debt by squeaking by that way, and has said several times over the years that this was his #1 mistake with that first job.

        1. Manic Pixie HR Girl*

          Yup. I fell into this trap myself with my first job. In hindsight, had I looked into roommates, I think I would have been happier (would have given me a social network/outlet I didn’t have) and it would have cost less. In the end it all worked out – no overly detrimental effects, and as it is the solo apartment I found was very affordable, so I don’t think my savings would have been substantial, in the end – but I do think I would have benefited from roommates that year.

          1. Koko*

            OTOH, I also moved into my own apartment the second I could barely afford it, and I basically treaded water on my debt from grad school for the first few years, paying enough that the balances didn’t rise but never went down. In year 3 I moved to a much cheaper apartment and got a little bit of breathing room. I took a second job in year 4 to both make extra cash and occupy me on nights and weekends to prevent me spending cash. Then in year 5 I got a big salary bump at my first salary review at a new job, and a couple of months after that I finished paying off my dead quit the second job.

            I don’t regret any of it. I’m an introvert with high self-esteem so I love spending time alone. When I lived with roommates I spent most of my time in my room with the door shut, and suffered perpetual guilt that my roommates probably think I don’t like them, when the reality is that I just really like myself. I’m also a standard deviation or two cleaner than the average 20-something so living with roommates was stressful in terms of the mess and not wanting to cook in a gross kitchen. (I pick up after myself daily and had roommates in the past who would let the entire house get gradually wrecked and then try to organize a group cleaning day where all the housemates set aside an afternoon to clean up together, which I super resented since *none* of it was my mess.)

            All the financial strain I had the first couple years, the working two jobs for a year thing, for me personally every bit of it was worth it. I valued being alone that much. That’s really the key – how much do you value being alone? Do you just think it’s the thing to do because you’re an adult, or maybe it would be nice? Or are you truly significantly happier by yourself than you would be in a group home?

            1. Manic Pixie HR Girl*

              This is a good point. I’m an extrovert who was an only child, so generally I prefer living alone* (because I am used to having my own space), but in this case, as I didn’t have a lot of social outlets at FirstJob, as I was new to town and it wasn’t exactly a bustling metropolis, I think (in hindsight) roommates would have helped ease that transition. When I was in grad school/First(Post Grad-School) Job, I had a number of external social outlets (a couple of intersecting extended circles of friends, hometown was closer, larger (but not big) city, school network, etc.), so living alone was a no-brainer.

              But yes, Tl;Dr, good to be self-aware and do what works best for you (if you can).

              *Spouse doesn’t count. It’s why I married an introvert!

        2. Brogrammer*

          The thing that bugs me about that is living with your parents while working your first job necessitates finding a job within commuting distance of your parents’ home. My only job offer out of college was over 100 miles from my parents. As it happened, I was able to find a cheap living situation until I got a raise, but if I’d had to move back in with my parents, I would have had to turn down the offer and keep job hunting in their city.

          1. Manic Pixie HR Girl*

            Yup. I ran into this problem. At that age, it would have been my preference if it was feasible.

          2. Overeducated*

            Yup. This worked brilliantly for my college friend’s who grew up in or near New York and Boston; not so well for those of us from depressed rural areas.

            I mostly loved living in a city apartment with 3 or 4 other roommates for half my $9/hr wage though. Then I got married, so I have never lived alone.

  7. Kristine*

    OP, I was you about 6 years ago (during the height of the recession), so hopefully my advice is somewhat helpful. It sounds like you have your list of cons about this place. What is your list of pros? Are there any? See if you can identify how this job might help you in the future. Does the boss seem like someone who would be a good mentor to help you start out in the field? Does the role/work have room for increasing responsibility and exposure to new developing new skills over time? If you can identify some benefits to this job then it might be working taking for 1-2 years to get yourself built up.

    I took a crappy job right out of college, the first one offered to me. The pay was low, morale was low, office was dim and dark and sad, and management was incompetent on so many levels. BUT it was a small company so I got to wear many hats and learn tons of new things (some of which I learned I liked, others I learned I hated), I had the greatest manager a newbie could have ever asked for, and was able to find a group of coworkers who banded together to keep sane. In all, definitely not the worst job I’ve ever had, even if it was objectively crappy.

    1. Shelly*

      I think Kristine has a good point here. When I was right out of grad school I took a job below market rate and in a place I did not want to live, because it was in my very hard to find a job in field. Part of what helped me decide was looking very carefully at my options and making a list of pros and a list of cons. My Pros barely outweighed by cons and it was the right choice.

      The last thing I will add is this: Almost ANYTHING is survivable for one year. This is a job. Not a life sentence. I know it sounds like a huge deal right out of school, but jobs are just jobs. They have perks. They have downsides and you can always leave. You’re not chained to your desk the moment you accept an offer.

      1. Mints*

        Yeah, first jobs are (from personal / friends experience) usually a 1-2 years. After two years it’s like an amazing bonus that you actually liked your first job enough to stick it out.

        I was at mine 20 months and I hated it by the end, but I was paid well and it was easy and I learned skills on my downtime. I was able to learn enough that I was a lot more marketable and got a way better job two years out of school

    2. Engineer Girl*

      In addition to pros and cons you should also have deal breakers and bonus points.
      I also want to echo on roommates. They are are a huge help if moving to a new area. That said, a bad roommate is hellish.

      1. nonymous*

        If OP isn’t experienced in roommate matchups, she may have a better experience renting a room in someone’s house. My friends who do this (the homeowners) are generally a conscientious, frugal crowd and certainly they have more “skin in the game” for keeping things clean that a traditional lease share. To be clear, look for a “roommate” situation with the owner on-site (sometimes people rent their basement as an apartment, and that doesn’t seem to work as well because the sound/privacy expectations can be unreasonably high). It’s also reasonable to ask for a 90 day trial period before committing to a 6 or 12 month lease.

        1. Temperance*

          Just my .02, but I would actually advise against moving in with the property owner. In my experience, the owners tend to be more busybody-like and demanding about cleanliness, upkeep, etc. than traditional roommates. I’m a super introvert who hates authority, though, so YMMV.

          1. Stephanie*

            Yeah, I wouldn’t say I’ve had bad experiences, but the power imbalance can be tough to navigate. When I’ve been in that scenario, it never really felt like my “home.”

          2. Teclatrans*

            Yeah, I see the sense in the argument, but the two times I subletted from homeowner/primary renter, there were a lot of restrictions of which rooms could be used when, how the kitchen got used, etc. One experience turned out fine because boundaries were clear ahead of time ans I know what I was getting into, and moved out as soon as I wanted to do things like invite friends over and actually use the living room. The second experience was pretty awful because the homeowner didn’t know his boundaries, and tried to offer to share the whole place with no restrictions, leading to his being snippy and paranoid and it was awful.

            I had several other roommate setups, either college friends or folks found in the local paper (I am an Old), and they were fine. Some issues regarding how often cleaning should happen, but married life hasn’t been much different, TBH.

    3. BPT*

      Regarding how this job could help you in the future, there’s something I always do when I’m considering a job. Go to LinkedIn, go to their Advanced Search, and put the name of the company in the “Past Company” line. I then look at the people who have moved on from the company, and see what they’ve done since, and what the experience in this role can set you up for. (I believe it also includes people who are still at the company but had previous roles at the company as well, so they moved up, so you can see career projection through that.) It’s probably more useful at smaller companies rather than ones with 500+ people (that’s a lot of LinkedIn profiles to go through), but it can really give you a sense of how long people stay, where they go, and what you can expect. Are entry level employees usually moved up after a year? Do they stay in the same role for five years? Does everyone leave after six months? This can probably give you more information than just looking at Glassdoor reviews (which, while helpful, can skew negative.)

  8. AndersonDarling*

    As Alison mentioned, it’s hard to tell whats going on through the perspective of the LW. Is low salary mean minimum wage or just the low end of the market pay range? Is “sacrificing a comfortable living situation” living with 4 roommates in a roach infested apartment in a bad neighborhood, or is it having an apartment that isn’t like the one on “Friends” or “Big Bang Theory.”
    I think everyone has a wake up call with their first job. But after two years, you can bounce off into a better job and that is when things really begin to pay off.

    1. Manders*

      This is a good point. I’ll also add: OP might want to consider speaking with a therapist about the kind of environment they could thrive in. I know some people who crashed and burned early in their careers because they moved far away from their friends and family while they were dealing with a mental health issue–but I know others who thrived because a new friend group and a new routine were exactly what they needed. Mental illness can really mess with your judgement about risk and reward when it comes to changing the status quo.

  9. Archie Goodwin*

    “The really important thing to factor in here is what your other options are. If you’re easily getting interviews and are confident that you can get other offers, it makes sense to walk away from a job that you have serious doubts about. But if you’ve been searching for a while and this is the only offer that’s materialized, that might be the market telling you that you’ll need to take it in order to build up a work history that eventually will give you more options. ” – this is a key point.

    My first job after college – which it took me six months to find – was, while not terrible, low-paying and not at all what I’d hoped to get with any sort of a degree. Nevertheless, I stuck it out…I was there for about a year and a half. I learned what I could, and worked with whatever was thrown at me. And once I’d updated my resume with my new skills and posted it online, I got a hit in under a week that led to my next job, where I stayed for over five years and was generally happy as an oyster. (I’m from Virginia…we don’t do clams here.)

    That said…how’s your gut? Has it led you astray before? If you have a gut feeling that this place would be HORRIBLE NO GOOD HORRIBLE GOT TO GET AWAY, then I’d pay attention to that. The gut doesn’t always work – mine’s screwed me up once or twice before – but generally speaking it’s a good indicator if you let it be.

  10. Sibley*

    Honestly, I hand the crappy stuff off to the new staff when I can. It’s a perk of being more experienced, and also appropriate to the new person’s abilities (and pay rate) sometimes.

    Real life example: I need someone to check 1200 email addresses and correct them. The newest person on the team is doing this for me. However, as boring and pointless as it seems, I actually DO need those emails checked, and if she gets it wrong, there will be consequences. She won’t see them, but I and pretty much everyone else will, and I’ll need to deal with it. So it does matter. It’s also dull, tedious, and boring.

    However, I’m polite, professional, and friendly to her. Just because I’m dumping the worst task imaginable on her, doesn’t follow that I’ll be nasty.

    1. Lovemyjob...truly!!*

      Yep! Years ago one of my tasks at a job was to check that all of the brokers affiliated with company had completed a specific training. There were literally over 100,000 brokers worldwide associated with the company. It was incredibly dull but necessary as the company approached year end. I was the newest employee and this necessary but dull task was left on my desk. I became amazingly proficient with that project and discovered a faster process that people who’d been there for years hadn’t realized existed. Eventually I was training new staff to assist me with this project. Yes, I still had to work on that project but by then I had started thinking of it as “my” project. It was hard to let go of it when I was eventually moved onto a different project where I could use my skills.

    2. Hellanon*

      Yep. First job out of school was a real mix – retail, with all the nights and weekends that implies, but also excellent formal sales training and a chance to put a very solid first year on my resume. Second job had lots more good stuff but also involved taking out the trash – literally. But second job got me third job, which also got me to Europe… start *somewhere* is the point, and get as much out of it as you can, and then move on.

    3. Kyrielle*

      I have been known to apologize to the new staff when handing them the most tedious jobs. They really did need to get done, but…the worst I ever did was (mumblety years ago) when we were converting hard-coded messages to something the user could change in the database.

      One new staff member got to go through all the code looking for every string (searching for ” basically) and determine if it was user-facing and needed to be replaced with a call to the new functionality.

      Every. String. For anyone who has ever read code, a) I presented why this was needed and useful and important, and b) I apologized, because seriously, the idea of doing this…ugh. It was not a small system, either, of course.

      1. Princess Consuela Banana Hammock*

        This is really similar to one of my first jobs, which included cleaning/debugging code and doing quality checks on our data entry (i.e., writing code and then doing a hard copy check against a sample). It was tedious, but it was also so helpful for understanding certain elements of data processing and research design. Which is to say, don’t feel too bad, Kyrielle :)

      2. paul*

        Heh. one of my first job duties was cleaning up formatting in our database. I actually found it soothing in a way.

    4. JustSomeName*

      I’ve never considered tedious tasks to be in the same category of bad jobs. If anything, being handed confusing, difficult tasks without proper training or sufficient experience is the problem. For me it’s always been the manager that would define whether or not the job was bad, and coworkers to a lesser extent.

      1. Princess Consuela Banana Hammock*

        I think the kind of tedium depends. There’s tedium where your mind can wander, and that can be fun/aight. Then there are non-tedious jobs that are mentally engaging. But tedium that requires a great deal of focus/engagement can be the worst, depending on what the assignment entails and the employee’s personality/strengths.

      2. sstabeler*

        getting tedious tasks is fine- it really is “paying your dues” since you are doing something dull, but necessary. where it becomes a bad job is when the new guy/gal is made to effectively be a servant/slave to the rest of the workers ( for example, take my last job- when I was the low person on the totem pole. At the builders merchants/diy store I worked at, out back we had a saw table for cutting wood to the length a customer wanted. It was my job to clear out the collected sawdust from underneath the table, and I never resented the job ( I even quite enjoyed it, and took pride in getting it as spotless as possible.) however, had they tossed (say) product packaging on the floor and amde me pick it up, I would ave resented that.)

    5. Hillary*

      Yep. Some of the dull tasks are also stepping stones, even though they don’t feel like that. One example, coding and approving bills can be boring and repetitive. But if you take advantage of it you can get a picture of a lot of activity you’re not otherwise exposed to as a junior person, and it’s vital to the company’s operations. I try to balance it out by including the junior person where I can – they sit in on lunches and meetings, I talk them through my negotiation thought processes, and I try to expose them to the more complex work that comes later in their career progression.

    6. Teclatrans*

      So, I am at a different point in my career, but that sounds kinda fun. I would have rocked that task. (My point being, I guess, that some drudgery will feel more awful depending on your skills and temperament.)

  11. MissGirl*

    It would be great if the OP could talk to someone in their field in the new city to get an understanding if their expectations are in line. I wish I’d been able to talk to someone in my field before I graduated. Knowing the realities would’ve helped me curtail my expectations. When I got offered $2 more an hour than I was getting at my part time warehouse job, it wouldn’t have been such a shock. It might’ve also convinced me to find a new career a lot sooner, or maybe not. When you’re 22, you have stars in your eyes.

  12. LisaLee*

    I got a job right out of college that didn’t pay my bills. I had to take another part-time job to make ends meet, but here’s why I did it:

    -I got to move to a city I wanted to live in, and it can be very difficult to find a job that will hire a far away candidate, especially if you want to move somewhere where there’s already a lot of jobseekers.

    -It was in my field, which was very saturated and a hard place for a new grad to find a job. I want to eventually go back to grad school (essential for a good job in this field) and this was a great stepping stone.

    -Personally, my mental health is better when I don’t live with my parents, and this job allowed me to get out of the house. I was far less depressed working long, crappy hours in a place of my own than I was with lots of free time in my parents’ house. This may be different for you.

    I’m not going to lie, the hours really sucked until I managed to wrangle a single full-time job that paid well, and my social life suffered. I had to live with a roommate, but I don’t mind that so much. I prioritized living somewhere where I could use public transit instead of owning a car. Only you can decide if the benefits outweigh the costs.

    1. Emi.*

      On another practical note, in case you do take the job and move, WalkScore lets you search for rentals within a certain walking-distance of bus and/or rail stops. They don’t have an exhaustive listing (who does?)–in particular, I don’t think it includes private listings and roommate deals like you see on Craigslist, just apartment buildings run by companies–but it’s a place to start.

      1. Jessesgirl72*

        Even if an address isn’t o walkscore, you can put the address into other places (zillow, even) to get a walkability rating.

  13. shep*

    I struggled to find ANY position after graduate school (overqualified education-wise; underqualified via experience), so I would’ve jumped at any offer toward the end of my job search.

    I did jump, in fact, and into an ill-fitting position in a field I had neither trained for nor had any interest in, but it served as a springboard into something I enjoy.

    So I would also err on the side of caution/your parents’ recommendation/Alison’s advice, especially if the offer is for a position in your field. It may not be ideal, but more likely than not, it’ll serve you very well as a resume-builder and a stepping stone on the career path you want to follow.

  14. Teapots Inc HR*

    I’m always baffled at these questions. If you’re coming straight out of college, why are you expecting median salary with zero work experience? First jobs very much are entry level and are basically there for you to get real work skills, learn interpersonal relationships and office culture before getting promoted or moving to another company. This entitlement from new grads needs to be checked. I see it all the time in interviews and when calling to make an offer. If you don’t have the relevant work experience coming from college, explain why I should start you at median salary?

    1. Ask a Manager* Post author

      But she didn’t say she’s expecting median salary. It’s possible that this job truly is significantly under-paying for the work she’d be doing and the skills she’s bringing. Just as we don’t know if she’s right about that from the letter, we also don’t know that she’s not.

      1. Ask a Manager* Post author

        (I do agree with you that some new grads have very unrealistic expectations of what salary they should be earning. We just don’t know if it’s the case with this particular letter-writer.)

        1. LSP*

          Personally I think it’s shameful that a job that requires a degree, even at entry-level, doesn’t pay a living wage.

          1. Jessesgirl72*

            The job might very well pay a living wage but someone who has an unrealistic living standard would think it didn’t. It’s not really unreasonable for entry level jobs to mean you need roommates, especially in more expensive areas. However, a certain percentage of college grads believe their first job should afford them a swanky downtown apartment or 3 BR house. Alison did a really good job, in my opinion, of addressing that possibility without assuming the OP is one of those.

            1. Anon Two*

              I agree. I think, in all but the most expensive cities (such as NYC, San Fran, Boston, etc.) that most people can live off $12 an hour (assuming that benefits are included and it’s a full-time job). Now it might not be a pretty, but about 25K a year is enough to keep a roof over your head and food on the table, with a little extra to go and live life.

              1. snowball*

                boston is a little more expensive now – but I moved here 9 years ago and made about $28k/year and I did okay. Not great, but I could share an apartment, have cable, and go out every once in awhile. I didn’t get health insurance through work but Romneycare helped.

                1. Overeducated*

                  It’s a lot more expensive now. Single rooms in shared apartments in my neighborhood are 60-100% more than they were 9 years ago. $28k is still livable for someone in a shared apartment wit no major bills, IMO, but everyone hasa different situation.

            2. Engineer Girl*

              Because the TV shows all have that. And the people they see around them are living at a higher level (because they are at 15 years experience).
              We all had roommates when we started out. And we were all engineers.
              When you are less experienced and your salary is low then you have to make choices.

          2. sunny-dee*

            And also, not all degrees are created equal. If I got an art history / whatever-studies degree from Brown, I just dropped $150,000 for no usable knowledge or skills, while an engineering or accounting degree from Kansas State is way more useful for, like, $70,000 or less. My cousin has a job in banking and she just has an associate’s from a local community college, but has zero debt.

            The employers require a degree as a substitute for a general skills test. It’s a way of (maybe) ensuring that they get candidates with good written and spoke communication skills and the ability to research and complete tasks. That’s really it. (Aside from professionally credentialed things like accounting or nursing or law.)

            But an employer isn’t going to go, “oh, wow, you went to UPenn? Let me double your salary!” Because they don’t care.

            1. animaniactoo*

              Yes, but an employer who is requiring a degree to do a receptionist job (or slightly above with easily teachable skills) is basically shooting themselves in the foot. Reception and similar basic skills jobs do not require a college degree. And if it does you better be prepared to pay like it does, *not* a dollar above minimum wage and expecting people who went to the time and expense of going to college to stick around for that for longer than it takes them to find something else. That’s the kind of setup where nobody should be surprised that people are continually leaving after a maximum of 4 months.

            2. Elizabeth*

              I wish we’d stop assuming that degrees in the arts or humanities don’t come with “usable knowledge or skills.” They do, including the ones you list re: written/spoken communication skills, research, critical thinking, etc. I know it’s really easy to malign these types of degrees, but we do everyone a disservice by suggesting they’re completely frivolous.

              1. shep*

                Thank you. This. I think I understand what sunny-dee meant to say, but as a person with a terminal degree in writing (which is very usable and requires an exceptional amount of skill, but is admittedly undervalued in the job market), the phrasing above chafes.

              2. Anxa*

                I know unemployed engineers and art history majors with great jobs (that didn’t come from wealthy families, either).

                Plus, a lot of these comments come from an assumption that there are fail-proof majors. Nevermind that not everyone is cut out for the same program, but there are other reasons not to push someone into a field they have no interest in. Like the upcoming gluts and bubble bursts and geographical variations and discrimination within those fields.

            3. Triangle Pose*

              While I understand your sentiment and advice for OP, this isn’t always true but people love to say it. In my field, the school you attended absolutely matters and the salary difference between a local/regional institution and an Ivy or Top 10, even three years out of school is in the $40,000+ range. I’ve worked in a few industries where this holds true. A Wharton (Upenn) degree will get you a much better starting salary (and down the line salary) at a much better firm in ibanking, consulting, etc.

              1. Julia*

                Yes. And in countries like Japan, your university matters more than your major, and mine has opened me several doors. (Granted, I also speak English better than 99% of the Japanese population.)

          3. Princess Consuela Banana Hammock*

            I agree with you, although I do think some new college grads have unrealistic ideas about what constitutes a “living wage.”

        2. Lovemyjob...truly!!*

          I don’t think that all of these grads come to these unrealistic expectations by themselves though. :( I remember when I was in college they would have alumni come in to speak with us about the jobs they were working in the fields we were studying. Some of the alumni were recent graduates and they’d tell us what they were getting paid and some of those salaries were high. I know I graduated thinking I was going to be making a lot more money and that jobs were going to be more plentiful in my field than they were based on these college sponsored events. After a year of job searching I ended up with a retail position that ended up paying more than a lot of my former classmates were getting in positions in our field of study.

          1. Emi.*

            Yeah, and when you’re in highschool you get all these college mailings saying “Our graduates have an average starting salary of $67,000!” and things like that. (That side-eye was for you, CalTech.)

            1. Jessesgirl72*

              My husband’s starting salary WAS $65K.

              What he didn’t realize was that $65K didn’t get you very far in Silicon Valley in 2001 (or now- the recession actually helped us!) It sounded like a lot of money to a 22 yr old living in the middle of nowhere in Indiana (Purdue)

              1. Anxa*

                And that those averages sometimes don’t consider those who didn’t get a job in the field or ended up in paid internship or with part-time jobs.

          2. AndersonDarling*

            Yep. And all the marketing materials the colleges use, “After completing our program, you will be ready to step into a Senior Executive role!” And the schools really push their internships as preparing you for a career to make you think it will get you past the entry level jobs and into middle management.
            But that’s what schools do. sigh

            1. Shazbot*

              Or you get the experience I got, which was going to the college Career Office and looking through 10-year-old outdated materials that cheerfully stated that with my degree I could never hope to make more than $23K per year and should get used to the idea of living in a trailer (yes, it actually said that). And then having the career counselor give me a Myers-Briggs test, look at the results (my score covered half of the possible outcomes), shrug and tell me they couldn’t help me, sorry, appointment is over.

              College (and grad school) isn’t the same as job training or job placement, but it seems like they do a spectacularly terribly job of helping students translate what they’ve spent years learning into a path going forward.

          3. boop the first*

            This is why I ended up in a pastry arts program. I felt it was sufficiently “hands on, creative” type work to not hate it too much, and the Industry Training Authority claimed it would pay $21/hour to start. (Which, to be honest, is what our city claims to be “poverty line” wage, but it was more than double what I was currently making so I fell for it).

            Turns out there’s no jobs out there, and if there are, it’s in a small business that pays minimum wage. Even LESS than minimum wage because they never update their job ads. It’s been 6 years since the program and I’m still making minimum wage. In retail. In a union, even.

            1. Aglaia761*

              Former pastry chef here, you also have to be very detail and task oriented to be a pastry chef. It’s very much controlled creativity. You have to understand ratios and how adding even a bit more of one ingredient can throw off the entire recipe.

              The problem with programs like this is that unless you are in a large enough city/metro area or have instructors with a variety of backgrounds, you won’t get a true understanding of what it takes to make more than minimum wage or grow in your career. Very few restaurants employ a true pastry chef. Most of the work is in hotels or industrial kitchens. If you are super creative, you can start your own business, but a lot of places won’t let you do it from home. So you have to pay for licensing and fees on top of your business costs.

              Unless you work in a hotel, the life of a pastry chef is hopping around from job to job. Either following your chef or sous chef around, or sniffing out the newest restaurants to try and get in while they’re hot and you can get some reflected shine. Most of my instructors didn’t settle down into a long term job until 15- 20 years into their career. And long term meaning ~5 years in one place.

              If you truly do want to work in the industry and have fairly steady hours, working in a major hotel chain is a great place to start. You’ll get to perfect certain portions of your craft and still get to be creative every now and then

              I went to CHIC which was a Le Cordon Bleu affiliated school in Chicago. Once I got there, I learned that I would need to stage aka work unpaid for at least 6 months before any of the decent restaurants would even think to hire me as a pastry assistant. I didn’t learn that from the career services department, I learned that from my instructors who gave us tips on what they did to get ahead in their careers.

              The upside is that I got to work at some of the best restaurants in the city. I also got to go overseas and work for 18 months after I received my Grand Diplome in France.

              The downside is that I ended up taking an extra 40k in loans to cover my living expenses during the 8 months I worked unpaid and to cover the cost of my Grand Diplome. I’m still paying those additional loans off. It’s one of the reasons why I left the industry and moved into the field I’m in now.

        3. Mononymous*

          If only college/university major advisors could offer a sort of “here’s what is a reasonable entry-level expectation for salary/types of work assignments to expect in your first X years/generic theoretical career trajectory in this field” course for their students. Maybe some of them do, but mine certainly didn’t… I also know that’s probably very unrealistic for some major tracks, especially for faculty advisors who may not have themselves been in the field outside of academia for a while. And likewise, not all career paths involve college or having an advisor to begin with. I just wish I’d had access to that level of mentoring back in the day.

          1. Angela*

            Schools are beginning to require their career offices to post their first destination surveys. Ours reflects what I believe is a realistic salary range for the industry and the geographic area. However, I have gotten some side-eye from enrollment, because the numbers are not necessarily “impressive”. It drives me nuts to get pressure from the industry to make sure students have “more reasonable expectation” and then get pressure from enrollment to somehow increase the average salary to sound better. There are a lot of factors at play and really none of them is student entitlement.

          2. Rob Lowe can't read*

            I agree with this. I went to college intending to enter a field which I knew no professionals in, so the alums who came to Career Day and the department’s literature about employment possibilities were where I got all my knowledge about career options. And they sounded great! But what I didn’t realize until my first internship (junior year) was that you had to put in time in really low-level, low-paying work for years, and probably pick up an expensive graduate degree, before you even got a shot at those more desirable jobs. I feel lucky that I had the opportunity to intern (almost exclusively unpaid in that field) and learn that before I graduated.

          3. AcademiaNut*

            I actually did this for a grandson of a friend of a friend recently – a first year student very keen on transferring into my field.

            I gave a lengthy and very honest description of the education and career path, job prospects, types of work we do, pay, pluses and minuses, and reasons for leaving the field. Ultimately, he hadn’t realized that you need a PhD (there are almost no field-specific jobs with an undergrad only) and you generally need to move to another country at some point (the local job market is small), and decided to stay in his more practical original major.

            Although in my field, very keen undergrads aren’t generally going to listen to the negatives that drive people out of the field – the difficulty getting a permanent job, the toll of long hours and frequent international moves on family life and work life balance, the fact that you can triple your salary by going into industry. They all start out believing that they are smart and talented and hard working, and will be the ones to get a faculty position at a good university in their home country. Then, ten or fifteen years later, people look at their job and their lives and go “Oh, *that’s* what they meant!”

    2. Trout 'Waver*

      Jobs at all levels are about trading your labor for money. Entry level jobs are no exception. They’re not training programs for other jobs. They’re jobs.

    3. Murphy*

      I don’t think OP sounds entitled at all, at the very least because they’re specifically writing to ask what expectations are reasonable. It’s not unheard of that young/inexperienced people are taken advantage of. OP is doing research and asking good questions, which is a good thing.

      1. Mononymous*

        +1, I’m not sure how else OP could be expected to know what’s reasonable besides asking, except maybe trial and error, and who wouldn’t want to avoid making errors in their newly-developing career path if they can avoid it?

      2. Tyrannosaurus Regina*

        Seriously. This is NOT an example of some woefully Entitled Young Grad nonsense, not at all. The writer is trying to calibrate their expectations realistically.

    4. Anonymusss*

      I’m always baffled at these questions. If you’re trying to hire people with a college degree, why are you expecting them to work for peanuts? Entry level jobs that require a college degree are basically there so employers can hire someone with an at least moderate intelligence level, work skills at least somewhat related to job tasks, and the ability to work with others productively. This entitlement from employers needs to be checked. I see it all the time in interviews and when being called with offers. If you don’t want to hire people that can’t read and write above an 8th grade level, explain why a college grad should be started at an uncompetitive below-market salary?

      1. fposte*

        I don’t see college as the dividing factor there, though–why should anybody be started at an uncompetitive below-market salary?

            1. Fact & Fiction*

              The snark toward you was unnecessary, but I believe this was a tongue-in-cheek counterpoint to Teapots Inc. HR’s above post, e.g., from the prospective employee’s view rather than the employer’s.

      2. Engineer Girl*

        College is a lot more prevalent now than 30 years ago. So you shouldn’t have the same expectations.
        Only 26% of my generation had a bachelors. Now days 36% age 25-35 do.
        That means more competition for jobs which affects market rate.
        I’ll also state that liberal arts degrees always paid peanuts. If you want more money you get training in something that pays more.

        1. fposte*

          Yeah, it’s not entry into “the few, the proud” anymore. And sometimes the college degree requirement isn’t because the job tasks need your art history degree; it’s just a company/org culture thing (and a class signal to boot) in a job that elsewhere is competently held by somebody with no degree, and it’s priced accordingly because that’s the market rate.

        2. Princess Consuela Banana Hammock*

          I’m going to gently push back, because getting a liberal arts degree at certain schools does not pay peanuts. Twenty years ago, people saw a college degree as a vehicle for upward mobility. As more employers require them and more people pursue them, as Engineer Girl noted, the return on degree has changed (sidenote: it’s hard to determine how much of the decrease is driven by the recession and how much is driven by changes in higher ed enrollment and graduation).

          But from what I can tell, the social class benefit of going to college is that, at some universities, you develop networks that may open doors for you that otherwise would remain closed. So if you studied English at Harvard and were pretty integrated into your college campus life, you’re probably going to be able to find a job that pays fairly if you’re open-minded about the field you’re in (although there are certainly unemployed Harvard grads suffering from Rory Gilmore Syndrome). But those social benefits may not be the same for other schools, even “high-ranking” (eww) ones.

          Higher ed is basically going through a process in which there are two competing narratives about the purpose of college education. One says to treat it like a technical trade school and pursue a “degree that pays.” The other view emphasizes the benefits of access to research opportunities and critical thinking skills. While it’s true that certain fields currently pay better at graduation, all else equal, the return on a bachelors degree from “elite” colleges/universities tends to even out over 10 years.

    5. the_scientist*

      Contrary to what seems to be a widespread belief (here and more generally), there are some programs that do provide technical skills. I did a co-op program in my undergrad, and I got valuable work experience in pharmaceutical companies. If I had been looking for work in those fields after graduating, I would likely have been able to negotiate a slightly higher starting salary (although not the median, which OP didn’t mention anyway) because I’d already worked in commercial laboratories and had already done some of the required GMP training. A new grad who had only worked in academic labs would have technical skills, but perhaps not the specific ones required, and wouldn’t have that training.

      However, I went to grad school, and took a job that underpaid me when I finished. Using real numbers here, the approximate starting salary for an entry level position in my field, in my city, is about $57K (it’s an expensive city). I made $40K with no benefits or vacation in my first post grad-school job, but I took it because the alternative was a 6-month or longer job search. When I moved to a new organization 1.5 years later, I increased my salary by much more than 20% and I was STILL on the low end of the range at the new company. So it is extremely possible that the salary OP will be offered is truly less than market rate. That doesn’t mean it’s not worth taking the job; I did it and struggled for a year but because it was a small organization I got a TON of really valuable experience that has allowed me to advance quite quickly in my new, much larger organization. But the financial stress (especially not having benefits) was a major motivator in starting to search the second my 12 months at the first job were up.

    6. Kristine*

      My first job out of college literally covered my rent (tiny bedroom in a 4 bedroom apartment in a bad neighborhood) and that was it. I had a second part time job to cover groceries and my subway pass. No way to know if OP is in a similar situation but it’s not unheard of for companies to offer less than cost of living to entry level employees. How is it entitlement to ask to be paid enough to eat?

    7. Kyrielle*

      I wish I’d had a _more_ unrealistic expectation. I accepted an offer that was low because I was new to the field…yeah, I was underpaid. It was not fair. It did get corrected a few years later…correcting for its effect on my salary resulted in a 35% raise. Not a typo, and I was important enough to the team by then that they did it instead of continuing to risk losing me.

      However. While that straightened my salary history out nicely and is thus better than many people do at recovering from an initial lowball, that’s after several years in which I was making so much less than I really should have been….

      1. Hush42*

        This is what happened to me as well. I took the offer without negotiating at all because I was desperate to get out of the job I was in. 2 years later I was still making the lower than it should be wage that I started at. My manager gave me a 15% raise in March when I got a promotion- that raise brought me to where I probably should have been in my old position and then he gave me another 15% raise in September (of the same year) to bring me to where I should actually be. He also promised me another raise in May when I graduate (working full time and going to school full time sucks but it’s worth it). At that point I’ll be making 50% more than what I started at so like you my salary will have mostly recovered from the lower starting pay.

  15. BRR*

    The two things I am curious about are salary and working conditions:
    -For salary, like Alison said figure out the market rate. I know someone who expected their first job out of college to pay around $70K when they jobs they were applying for paid $35K-$40K. For a lot of positions, what you learned in college isn’t super valuable for an employer. Look at what other employers pay for the same position.
    -For working conditions, is this from glassdoor only? I would think about what you witnessed during the interview.

    Wrapping it up, you’ll have to weigh your options. The attractiveness of this offer will depend on what you have to compare it to. If you have gotten nothing else, that makes this a good offer.

  16. Caroline*

    Getting my first “real” career-type job was SO HARD. And it paid almost nothing. And I was way overqualified for it–but all the applicants around me were also overqualified.

    In fact, I had to take a pay cut from my “fake” job (non-career, seasonal) job. It was so hard to make ends meet, even living with a roommate, but it was the best I could do at the time. I’d been applying for jobs for months with just a handful of interviews, and this was my only offer. (It didn’t help that this was 2008, of course).

    The thing about first jobs is that their purpose is to be a stepping stone to something better, either within the company or to another company. You shouldn’t expect to spend more than a year or two at that level. So even if you hate it and need to live with a roommate and eat ramen noodles for lunch, you can view this as only temporary.

    My best advice to you would be to go forward with the interview process, and in the meantime, figure out what you absolutely need in terms of salary in order to live–research the cost of a roommate situation in the area, make a monthly budget for yourself that includes your loan payments, etc. and represents your basic needs like food, housing, utility bill, etc. That way when you negotiate your salary, you know exactly where the bar of “I won’t be able to make ends meet at this number” is. But if they are able to meet that number, I think it would be best to take the job.

  17. NK*

    It really depends on how you define “crappy”. Sure, your first job may not pay as much as you’d like, and the work/organization may not be all that exciting or interesting to you. But if you get the sense that the employees are generally unhappy, I’d give some serious pause to that organization, because I don’t think your first job has to be for a terrible organization. But, as Alison mentioned, the big caveat there is that it really depends on what your options are. You may have to settle for a crappy company to start if you’re getting very few bites on your resume.

    Also, particularly if you’re a woman (but really, regardless of gender), be careful of pay that is below market rate (not below what you’d like, but actually below market). Unfortunately unless you’re in Massachusetts, most companies still rely on prior salary as a basis for pay, and starting low can haunt you throughout your career.

  18. Trout 'Waver*

    I would take the public reviews with a grain of salt. The overwhelming majority of people don’t post them, so they’re hardly a representative sample of the company. And some are outright false or exaggerated

    1. AndersonDarling*

      And bad reviews could be traced back to one manager or department. If it is a large company, 75% of the employees could be happy, but the 25% working in one branch may be producing all the bad reviews.
      Still, you don’t know if you are applying for a job in the 75% or the 25%.

    2. Dan*

      Well… they need to be a data point that the OP considers. I’m not saying that OP should put to much emphasis on them, but the OP shouldn’t ignore them either.

      FWIW, I’ve looked at the glass door reviews from my last employer (small place, struggling) and my current employer (large place, rather stable). I consider myself to be reasonable, and I’d agree with the reviews from both places. The reviews from my former employer were quite accurate, and if someone took a job there (not like they’re doing much hiring anyway), they should absolutely head the warnings, and use that information to form questions about company health and prospects and all of that. To ignore that and claim “oh they were all disgruntled”? Caveat Emptor.

      1. Janice in Accounting*

        +1. The Glassdoor rating for my former employer is very very low (essentially bolstered by the fact that you can’t rate the place zero stars). While yes, they are mainly reviews from disgruntled ex-employees, there is some major truth in them and hopefully those give some job seekers pause about applying there.

        1. Lily Rowan*

          Agreed. I also find that they are often specific enough so that you can tell if the complaint is about something that will have an impact on your work.

          1. Kyrielle*

            Some of the “negative” glassdoor reviews about my current employer made me want the job more. The exact thing that those people disliked sounded like a lovely environment to me – and yet I can see how it would be horrid for other people with different desires/needs/tastes. Not all negative reviews are a bad thing. (Which is why I take reviews with almost no content as noise, and mostly-ignore them. I want to know _why_…it might not be a problem for me. Or it might be a really big one.)

      2. Engineer Girl*

        Content is important. “They’re all jerks” tells me nothing. That’s an opinion. “We have to work 70 hour weeks with no additional compensation” is useful data.

        1. Kyrielle*

          Yep. “Management has no clue” is also pretty useless, whereas “Management consistently fails to predict the market and significantly changes specs 3-6 months before delivery” is specific and potentially interesting. “This place is circling the drain” is useless and “The clients are interesting, the products have potential, but our entire management team has turned over twice in two years and new hires are left to get trained by people who have been here six months and didn’t get full training themselves” is…well, run. :)

          And “There are no advancement opportunities” could be “No one ever gets promoted here, but they’re happy to hire from the outside at a higher title; be ready to leave and come back.” Or it could be “This company values long-term retention and higher-level employees generally only leave when retiring; first chance at their position goes to current employees, but you’ll have to hang on a long time to have a chance at it, and you’ll need to go elsewhere if you want to move up faster.”

          Interestingly enough, both of those “no advancement opportunities” scenarios could appeal to some folks. But not remotely the same folks.

        2. Brogrammer*

          Is it weird that I find this sort of thing to be a fun game?

          “Management is terrible” vs “Management doesn’t give feedback on anything until it’s already too late to correct the problem.”

          “Compensation is terrible” vs “Pays only 40% of standard market rate for a Certified Teapot Repair Technician in a high cost of living area.”

    3. Mike C.*

      The fact that they aren’t representative isn’t important here – for that you’re need a random sample anyway.

      What’s important about those accounts is that they give you an idea of the sorts of red flags to look for and ask about.

    4. Triangle Pose*

      Agreed. If I look at my company’s reviews, there are a lot of lower level techs or customer service folks who raise issues and leave negative reviews (some may be justified). But I was looking at a corporate level role so it was not going to tell me a lot about the environment I would be going into.

  19. LSP*

    If you can get a professional job within a year of graduation, take it. I spent my first 3 years after graduation writing for a local newspaper, at the exact moment newspapers began to sharply decline. I loved the work, and made a few friends that I still have more than a decade later.

    Of course, it wasn’t all rainbows and sunshine. My boss was the type that couldn’t decide if she wanted to be my boss or my friend, and even accused me of faking a migraine(!) to get out of work (because she really had no idea of what a migraine actually was). She was manipulative and because she was so disliked among upper management, anyone who worked closely under her was painted with the same brush. I also couldn’t afford to live on my own on what I was making (for a job that required a degree!).

    It was tough, but I was young and understood that I needed to “pay my dues,” and in the end, I got a lot out of it. I took a year to teach English abroad, and when I came back, a contact I had made during my time as a reporter hired me on, and I worked with her for 3 years making DOUBLE what I had made at the paper.

    I agree with Alison that you shouldn’t put up with abuse or a salary far below market rate, but learning to keep your head down and get on with your work is a valuable life lesson. It’s been 10 years since I left the newspaper and I am making more than 3x as much as I was then, and I have a great manager and wonderful coworkers.

  20. MarketingGirl*

    I was in you a few years ago. I applied to over 400 positions and was getting further and further away from my graduation date. I was starting to panic and finally got offered a position a company I was wary about. I turned it down and I’m SO thankful I did.

    With that said, there were a handful of red flags that helped me make my decision. I was about ready to jump on anything that came my way but I knew deep down this was not the kind of position I needed to be in. I was told about the high turnover, walked into a large and virtually empty (and dark!) office at 10am on a weekday, listened to employees tell me it was “fun” to sell data, and had my future manager question my competence in ways that wasn’t appropriate for an interview. Had none of that happened, I probably would have accepted the job.

    What are your hard limits? Did they cross any of them? Can you put up with a s0-so culture for a year or two and use the time to breathe/explore other options?

    I’m all about gaining experience. Chances are pretty slim you’re going to find the “perfect” position right off the bat. If you can use this position as a stepping stone in some way (live at home and save money, pay down your loans, gain experience, add something to your resume, broaden your skills), I’d say go for it.

  21. Tiny _Tiger*

    I can’t really say that my first job right out of college completely sucked, but it was definitely a dead-end (no advancement or raises). Mostly I was just happy to not be working nights and weekends consistently anymore. I hate to say it, but if interviews are few and far-between and this place offers you something, it’s probably better that you take it. Unless the pay situation is completely unreasonable (i.e. there’s no way for you to cut any more expenses and still be able to pay bills and eat), then go for it. I managed to jump ship from my first job within 6 months to something (mildly) better and have since moved up within that company.

  22. babblemouth*

    My first job out of university was fantastic, but th pay barely covered food bills and rent (and that was in a tiny room in a tiny shared apartment). We’d been told to expect high wages out of university because we came out of a Very Renowned School… but this was 2009, and I quickly realised I needed to adjust expectations if I needed to get anywhere (tbh, I also think these expectaions would have been unrealistic without a financial crisis too). I went for the job with low pay that could buffer my CV, while some of my classmates tried to wait til a job with higher pay came along. I’m certain that out of all of us, I made the right decision. The only ones that did seem to get a well-paying job just out of university were those whose dad just happened to be CFO of a large company etc.

    All this to say: OP, as Allison says, there’s low pay, and then there’s low pay. If you can cover basic life expenses with that pay, take the job. Chances are, you’ll either get a pay raise within the year, or you’ll be much better positioned to apply elsewhere.

  23. OP*

    Thank you so much for the advice!! Unfortunately I’m travelling today so I won’t be around in the comments but I’ll definitely be reading everything when I can!

    I sent this question before I interviewed in person yesterday, and I now have a lot more information about the job. The interview went excellently, and if they offer me the job, I think I’m going to take it. The office culture doesn’t look as bad as I feared and the position offers good learning experiences.

    It’s probably true that my expectations for my first job were off base. This job pays below what people from my major (physics) typically make right out of college, but that doesn’t mean I’m above it, or that I should expect a better paying job automatically. It does pay below market rate, but, weighing all the factors of the position, that might be a necessary sacrifice and a trade-off I’m willing to take.

    Regarding my mental health and living situation, as a couple people have commented on, those are my highest priorities. I do include considerations for my mental health in looking for housing. It’s a high priority for me to be able to have a pet and to ideally live alone for example, both of which raise my cost of living. I’m moving to be closed to family, though, so I’m not overly concerned about having a support system. Now that I have more information about the position, I don’t think it will cause any big waves in my mental health. My biggest sticking point is being able to live as comfortably as possible on the lower than expected salary.

    Thanks again for your help, and I’ll definitely be checking back in later tonight!

    1. Dan*

      Glad things worked out — it’s hard to turn down a job when you don’t have many options. I was going to write in and say that market rate is whatever you can get someone to pay you for your services. The absolute best way to see what you’re truly worth is to get two offers. I’m not necessarily saying you should play them against each other, but if they’re similar, then you’ve got a darn good idea what your pay should be. If you only get one offer? That offer may or may not be underpaid, but it’s hard to know because you couldn’t get a second offer.

      If the glass door reviews concern you, there are ways to discuss them at a high level with management. Pay attention to how management addresses them — if they blow you off, you should be very cautious. Same with them attributing them to “disgruntled employees”.

      I think another takeaway is that one should reserve final judgement until after the interview. How do you gel with the interview team? If you get along well and they will be in your chain of command, that’s certainly a positive. I’ve passed on jobs where the interview team and I didn’t establish a good rapport.

    2. Just an idea*

      You might want to ask your mental health provider about writing you a prescription for an “Emotional Support Animal (ESA)” since it sounds like you have had some mental health issues & either have or would like a pet. If you have an ESA, you don’t have to pay pet fees for your rental, which would also help your budget.

      ESAs are different than service animals because ESAs do not have public access rights (stores/restaurants, etc – but they can fly in the cabin of the airplane).

    3. BRR*

      I just want to comment on salary again. What people earn for their major isn’t a great metric to me because it’s going to largely depend on what type of position you’re applying for. When talking about market rate, I think another perspective to think of is your market rate. I don’t know how you rank as a candidate, but based on a number of factors there will be a salary you can command.

      1. Triangle Pose*

        OP did say that she was looking at what people from her major make “right out of college” – that’s not a bad metric for her to use. Sure it depends on the level of the position they are applying for, but if everyone is right out of college, they are all going to be pretty similar entry level roles.

    4. Meg Murry*

      It’s good to hear that the jobs seems better than you thought it would. However, since it sounds like this would be your first full-time job, I’d suggest you work with your parents or another more experienced person you trust to make sure your budget is realistic for the salary discussed. Especially with regards to what your actually monthly take-home pay would be vs what the offered salary is. For instance, will your parents keep you on their health insurance, or will you have to start paying for your own coverage? Or if you aren’t living in the same area, will their insurance actually cover any doctors in your new area, or will you need to get your own plan for that reason? Are your estimates including enough taxes being withheld? If things go really south with this job, can your parents offer you any kind of safety net, or do you need to try to live much more below your means to start a robust emergency fund? Are your budget numbers for housing realistic, or are they a theoretical fantasy number and the true going rate for apartments that allow pets and are available now is way higher? What about the cost of utilities, transportation, etc?

      I’d suggest looking at a take-home pay calculator (lots available online) and making a robust budget before you sign any leases, etc. But I hope this new position can work out for you, even if it means living a little tighter than you’d like initially.

      1. Government Worker*

        +1

        Reading this thread has made me think about the range of perspectives that people have about how much they need to live on. I and many of my friends moved to big, expensive cities for our first jobs, and I can’t think of anyone who lived without a roommate (or several), and very few people had cars. Living alone or saving money might have required living in a pretty unsafe area, having an apartment with (more) vermin, or other unacceptable tradeoffs.

        On the other hand, I have a relative who was a teacher in a small, remote midwestern town just out of school Salaries were lower, but she lived alone in a 2-bedroom apartment that cost literally a quarter of the rent on the much smaller 1-bedroom my spouse and I shared in a big coastal city a couple of years later. But she wouldn’t have been able to manage without a car, whereas I didn’t own a car for years.

        So, OP, do some research about the specific area you’ll be living in to figure out what it will actually cost before deciding whether you can afford to take this job.

    5. Emilia Bedelia*

      So I’m also a fairly recent grad, and one thing that I really did have to readjust my expectations on is how well my skills would translate to my job. My degree is relevant to my job, and definitely prepared me for it. But, I don’t really use my “hard skills” – rather, the things I learned in school provide me the background to understand what I actually do. As an example, I know a lot about the technical specifics of glazing a teapot, but in my job, I write reports about how glazing quality was evaluated- not something I actually learned in school, but knowing how the teapot was glazed certainly helps. Going into my job search, I probably gave a lot more weight to my “skills” than they warranted. Your abilities as a fresh grad are just not as strong as more experienced people’s, and if they only need a new grad level of skill, they’re going to pay at that level.
      Unless you’re applying for a lab position or a position where those skills are, in fact, the backbone of the job, using your knowledge in conjunction with other soft skills (writing, critical thinking, problem solving, etc) is more important. I don’t know what jobs exactly you’re applying for, so I may be off-base here, but I’d suggest pivoting your view to focus less on “hard skills” and think more about your general knowledge base and thinking/communication skills.

    6. HR Pro*

      OP, glad to hear the interview went well and you feel better about the job!

      I’ve been in HR for over 15 years, in a medium-large sized East coast city with pretty high cost of living. I don’t know about jobs for physics majors, but I can tell you that most entry-level jobs for people with Bachelors degrees in our city are not paid enough for someone to live alone — unless you have special circumstances, like a parent who is contributing to your finances and no student loans, or you’re living with parents, etc.

      I had 3 roommates for several years when I started out in the working world (in the same East coast city I live in now). I was paying student loans. All of my friends were in the same situation, except for the ones who were living with their parents. I had to be very careful with my money. My only “luxuries” were to pay for yoga classes and to pay for a therapist which was mostly not covered by my insurance. I had a very old car. I never got into a financial crisis, and I was able to contribute to my 401k, but I was not at all extravagant. I clipped coupons, shopped sales & second-hand stores, etc. I would imagine this should be your expectation — unless, like I mentioned earlier, you have other circumstances.

    7. Engineer Girl*

      Pet friendly rentals are extremely hard to find and expensive. I have a suggestion – “no pets” usually means no cats or dogs. How about a rat or a guinea pig?
      Also consider a roommate that works opposite shifts. You’ll only see each other on weekends.

  24. Laura*

    One other thing, I’ve noticed that a lot of online reviews tend to be from people who left rather than those who are still employed at the office. So 200 people might be happily employed there, anther 50 have moved up and on, and 10 were let go with an ax to grind. Guess which people are the most likely to post a review? I would take them with a grain of salt.

    1. Leatherwings*

      Eh, that can sometimes be true but I hold that a ton of consistently negative reviews are a red flag.

      1. Triangle Pose*

        I think it really depends. Especially if the company employs a lot of X type of employee and you’re going in to a Y role. For example if you are interviewing for a corporate level accounting job at a huge telecomm, customer service. or retail organization for example, you’ll probably see a lot of negative reviews from install techs, customer service agents, etc. But those negative reviews are not really going to give you information about the environment you are going into. At most, it tells you a segment of the employees has high turnover, but not much else.

    2. Two-Time College Dropout*

      And if the company has multiple locations and departments, then someone’s experience in the spout design department at Teapot Depot’s Anytown office may not be that relevant to the handle production position at the Somewheresville location that you’re considering.

  25. Kitkat*

    My question for you is: if you take this job, and, worst case, it’s absolutely horrific, what would you do? Could you quit and move back in with your parents? How stuck would you be? Of course this depends on the mental health issue, but if you feel resilient enough that if worst case scenario, you could just quit and leave, then I think the upsides outweigh the downsides.

    I lived on $17,000/year a couple years ago in DC (a very expensive city) in a basement room without windows and no access to a kitchen doing a very draining job. And yet, most days I was THRILLED. Thrilled to be in a city I loved, to feel completely independent from my parents, to be earning money and deciding how I was going to spend it, to be making strides, even small ones, in my career. I latched onto the things that made the job worth it, and a couple years later am getting paid twice as much in a job and organization that I love. It seems like a lot of us have stories like that, of having made it through Crappy First Job to something better, and so while I do think it’s unrealistic to assume your first job will be amazing in every way, it’s also unrealistic to think that you’re absolutely doomed to Crappy Jobs forever and ever!

      1. Stephanie*

        Yeah, I wish there were a way to incorporate that into a resume…because that’s some impressive budgeting.

  26. jenny*

    I am you. I moved to a new city for my boyfriend’s job, and left my first job out of school at a really amazing company to do it. I had taken one of those really crappy, low-paying, stepping stone jobs at Cool Company and left before my 1 year mark and before I could move on to anything interesting or that would set me up to launch a career. So, my experience was therefore useless and I had to completely start over in a new city with no contacts. It was awful. It *IS* awful. I job hunted for 6 months and finally accepted a job I hate at a place I do NOT fit in out of desperation. The pay is okay, but I literally sit and do nothing for 40 hours a week. The title is embarrassing for my education level and it feels like a waste of my time. I’m taking it as a sign from the overlords to enter into a professional program and use my 40 hrs of nothing a week to study for my entrance exam. I don’t know how this position could POSSIBLY launch me into any kind of career, so grad school it is. It’s been a disheartening couple of years since graduation and I have nothing to show for it. I hope you get along better than I have.

    1. jenny*

      Also, I should mention I took a part time serving job on the side at a fun restaurant for a couple of reasons: Unemployment is expensive and I have a lot of debt to pay off, to be social and around young fun people instead of all the catty nut cases at my day job, to save money for school application expenses (prep courses + test fees + application fees + reserving a spot + new computer = $$$), and because I don’t want to associate “work” with “despair” :( good luck

  27. caryatis*

    OP, know that you can live on less money than you probably think. If you haven’t already started learning about personal finance, this is a great time to do it. Minimizing rent, transportation, food, and clothing costs will put you way ahead of most recent graduates and allow you to save on any salary.

    This is not to say you should take the job–just FYI. Oh, and sadly in my experience every first paycheck ends up less than you expect.

    1. Emi.*

      My first direct deposit was more than I expected (I just didn’t do the math, despite its being a data analysis job) and I was super happy (think Calvin with his first allowance), and then when I got my earnings and leave statement with all the deductions, I was like “….Oh.” :P

  28. Barney Barnaby*

    Just throwing this out there: the entry level salaries that I’ve seen on websites seem to be way out of whack with what people are actually paid. The problem is that they are people who have gotten jobs in the field (which even with technical fields, isn’t the case), often at large companies (again, not the usual situation), and who report them (which may not be the usual case).

    As but one example, there was a study that came out showing that kids from Fairfield were paid more straight out of school than kids from Yale. If you examine the data, you see that the term “representative sample” wouldn’t exactly be what is used.

    If you’re in a technical field, talk to technical and scientific temp agencies. Temp to perm is a big thing in those fields, and even if you “only” get a temp job as your first role out of school, it will put you in a good position to build skills and get something better.

      1. Barney Barnaby*

        Nope.

        There were approximately a half-dozen people from each institution, or thereabouts (one might have been six people, the other, twenty or so), who answered the survey.

  29. Allison*

    I was in a similar position about 4ish months after graduation, so September of that year. I too took a job that paid less than market average, and less than I’d been looking to make, the job was outside my field of study and I’d heard bad things about the office. I took it anyway, but I lived with my parents while in that job because it didn’t pay enough for a decent life in the city. I didn’t move out until my next job, which was abut 10 months later after the first job didn’t work out.

    I’d say if you could stay living with your parents, take the job and see what happens. If you’re looking to move once you start getting paid, find a job you’re confident you can stick with for a good long while at least.

    At the very least, you should be asking questions that address your concerns in the interview. Ask how long you’d be at that salary before you could reasonably expect a raise. Ask about office politics. Ask about hours. Reviews are helpful, but take them with a grain of salt. Sometimes they’re true, sometimes they’re exaggerated, sometimes they were issues a long time ago but have been resolved since then.

  30. MashaKasha*

    My first out-of-college job was in another century and on another side of the ocean, so doesn’t apply to the OP’s situation. My first American job, though. In hindsight, I think I should’ve continued with my initial plans of going back to school instead of taking it. It was extremely low-paid (20K for a programmer job in 1997, and we were a family of four living on that money, until my husband found a job six months later), but that was not the problem. Problem was, it was a dead-end job. The skills I got there were so obsolete, it was impossible to find anything else by putting them on a resume. I was able to get out, but not proud of how I did it – my married boss, who’d harbored a dream of one day having an affair with the married me, found another job and brought me in. That second job gave me better, more marketable skills, and after eighteen months in it, I was in a far better position to look for work (and get away from the love-stricken boss, geez!)

    So my advice is: low-paying – OK. Dead-end – NO. It has to be a stepping-stone kind of job.

  31. Girasol*

    Office politics aren’t a bad thing for a first job. My first job was “crappy” in that way but I learned a lot that college could never have taught me about how the working world works, and was better able to navigate it afterward. I was paid barely enough to live on but it was so worth it. If you consider that first job as the college senior course “Challenges in ” instead of thinking of it as how your life will be forever, you can get a lot out of it.

    1. Girasol*

      Ack, I keep forgetting what characters blank out in comments. I meant “Challenges in Field of Your Choice.”

  32. Alton*

    With regards to the mental health aspect, I would encourage you to build a strong support system and make sure to take care of yourself. Even a really good job is going to be a big life change, and it can be more challenging sometimes to keep a healthy sense of perspective when you’re coping with something bigger.

    When I got my first post-graduation job, I really liked it in an abstract sense and I knew that it was a good job, but my loved ones were remarking that they were worried about me because I seemed anxious and unhappy. I couldn’t see what they meant, really, but eventually it got to a point where I realized that I was feeling a disproportionate amount of stress and emotion to things that weren’t a huge deal. I think the fact that I struggle with anxiety was making the transition period harder for me. But I made sure to take care of myself, and I was really careful to keep things in perspective. Over time, I’ve become a lot better at weathering the sorts of routine stressors that can come up in a job, and disconnecting when I’m off the clock.

  33. ilikeaskamanager*

    I like Alison’s answer. I would add one more thing. It is an important rite of passage to adulthood to become self supporting and you have to start somewhere. And I can assure you that you WILL NOT make what you think you are worth in your first job–I have never yet met anyone who did! As long as Mom and Dad pay the bills, Mom and Dad call the shots.

    1. Julia*

      Not true. I moved to Switzerland for a job and make more money than I thought I would and even more than I asked for. Granted, things are expensive here, but I am frugal and often shop for clothes or electronics when I visit my home country, and my parents pay for my flights home because they want me to visit several times a year, but even then, I am saving about a third to fourth of my salary every month. I realise this is not too common and mostly just luck, and I also realise that not everyone can just move to find a job – I certainly wouldn’t have if it weren’t for my partner going to school here, and I won’t stay here much longer now that he has moved to the next country, but I don’t think telling people that they should NEVER expect to be paid well straight away is not helpful either.

  34. fwong*

    OP, have you considered looking into staffing agencies? Scouring for jobs, writing multiple letters, etc can be exhausting and finding an agency that vibes with you can really take the weight and stress off. Mine gave me temp assignments while also searching for the right job for me. The employer is the one paying them so there’s not much for you to lose. I didn’t have to write cover letters, the recruiter set up the interviews for me, and when it came to accepting an offer, they were the ones who negotiated my pay (which ended up being higher than the company initially offered!). Companies that employ temps are also usually understanding when you tell them you’re looking for a permanent place and some even use recruiters exclusively so there’d be positions that would open to you that you couldn’t find otherwise.

    I was extremely lucky with my agency (they focused on employment for women), but I believe you’d be able to find something! Good luck.

    1. Misteroid*

      Yes! I don’t know if this would be helpful for OP, but it certainly was for me. I graduated in May 2015, and could not find a full-time position for over a year. I had a job, but it was the same job I’d worked in college, with 12-20 hours a week, mostly on weekends. (I held out because it was a sure thing, and also because I thought I was going to get promoted. It didn’t happen.)

      Finally, after a lot applications that went nowhere, I signed up with a temp agency and told them I was looking to do temp-to-hire. They found me a place that was great, used a bunch of my skills, and paid me more than I was previously earning. And if my new job had sucked, I would have felt comfortable going to the temp agency and letting them know I needed a new placement. It was the best choice I could have made, and I only did it because of AAM.

  35. Jill*

    The OP says that they are “still in the interview process.” So, OP, now is the time to ask more questions and flesh out whether the online reviews you’ve read mirrow what the hiring manager has to say. Other responders seem to be focusing on the idea that most first jobs aren’t ideal but can help you in the long run. But you’ve been in treatment for a mental health issue. Yes, a low paying, not quite what you want to do kind of job can help you as a new grad in general, but not if the job is so sucky that it knocks your mental health progress backward. So if you have the opportunity, ask more questions about the culture of the office, the workload expectations, employee turnover, management style, etc. The answers will hopefully give you some insight as to how you could handle the job in relation to the work you’re doing to improve your mental health. Best wishes to you!

  36. OP*

    I want to thank everyone for their advice and for sharing their experiences. It’s very helpful to hear from others since I don’t have experience of my own to fall back on.

    I’m especially thankful for the people who had advice on figuring out my own value in the market, and that I will probably not be paid what I think I’m worth starting out. I was basing my salary expectations on what physics majors typically make out of college, what my friends and family make, and what salary info I could find on the field I’m going into (data analysis). I failed to account for the fact that I’m applying outside of physics, that data analysis is a ridiculously broad field, and that my own skills and experience, while adequate for what I’m applying for, probably fall behind what others are able to offer.

    It’s also helpful to hear how others have been able to make do with low salaries. My cost of living expectations may also have been a little unreasonable. I’m definitely working with my parents to create a realistic budget.

    I don’t want to give the impression that I’m only looking at the cons of the situation. Overall, I’m excited to get out into the real world and be an independent adult! After interviewing in person, I’m excited about the prospect of this job as well. I just want to make sure the downsides are reasonable and that I can live with them.

    1. Anon Two*

      You seem to have a good head on your shoulders.

      What you are experiencing, at least in terms of salary, is very common. It’s very easy to use friends and family as the yard stick without taking other things into consideration (like years of experience, the type of organization that the person works for, or even if the person just got lucky), and then shoot yourself in the foot. And for all the talk about knowing the market rate for your skill set, I find this to be very challenging, and I have almost 20 years of experience!

      However, once you get your foot in the door, and you gain some experience and have some work related accomplishments it will be easier to find a higher paying job. My first job paid me pretty terribly, but I got a 40% raise for my second job. For that first job I would encourage you to look for an organization that is going to give you an opportunity to really learn the skills you need to move forward.

      My first job I brought home about $1000 a month. My rent was 50% of my take home pay. But, I managed (and it was a pet friendly apartment complex, so I had a dog!) and I am sure you will do as well.

    2. Moonsaults*

      I want to also point out that in the interviewing process, it’s easy to get overwhelmed and kind of scared about the prospect of working somewhere you’ve never been before. You don’t know these people, just what you feel and the reviews you get to see that are posted places.

      The important thing to remember that you are not going to be trapped anywhere. There is always a ripcord attached to you, you can bail if something was outrageous and you can often if you scurry away soon enough, even pretend the job never actually happened and you’ve been looking ever since college :X

  37. Lola*

    I took my first job because I was moving to NYC and my dad told me to take the job. I hated the job but after 3 months another job contacted me and I was at that job almost 2 years. You gotta pay the bills somehow but just because you take a job doesn’t mean you’ll be there forever.

  38. Nic G*

    One of my first jobs was working at Burger King. It was not a highly skilled, well-paying or prestigious job. I did learn a lot of valuable skills and had room to make terrible mistakes of a rookie employee. Also, I had fun! I worked with a bunch of folks my age or close to it and we goofed around while working hard. I didn’t like the job for the most part but I gained valuable experience helped lead to better jobs/positions. Keeping a positive attitude helped make the job tolerable, if not, enjoyable. Attitude is key.

  39. Green Tea Pot*

    My first job out if college was in a toxic environment run by a man I now realize was a sociopath. He made arbitrary decisions, cheated on his wife (she did the books) under her nose, and created problems among the staff.

    The duties weren’t bad, but I had no way of knowing that all job environments weren’t like this. I had personality disordered parents, and as a result, no idea of what was normal.

    I stayed because someone told me I had to pay my dues in the field. As a result, I was taken advantage of, given scut work and paid poorly.

    I now know differently, of course. And I’d advise anyone starting out not to setlle for a crappy job.

  40. ThatAspieGirl*

    No, your first job isn’t “supposed to be crappy”. My first job is good so far! “Entry-level” isn’t supposed to mean “abusive and horrible”.

  41. CodeMachine*

    I have been at my current job for about 17 months. It was the first offer I got and I needed money after graduating. The job offers very few benefits and quite a low salary(below market value). However,as a Junior, I am exposed to projects and responsibilities which other Juniors only dream about. This will certainly look nice on my resume. As an advice, try to find something better if you really don’t want this job, as you will end regretting it.

  42. The Expendable Redshirt*

    If two thirds of your pay check go to student loans and bills, that may be a sign to move on. Ideally, debt should take up no more than 15% of your income. Housing, utilities and such should be no more than 35%. (Transportation 15%, Life stuff 25%) And yo, 10% to savings! If those numbers are way off kilter, and your finances are unlikely to decrease….. Your current job may not be sustainable.

Comments are closed.